2019-2020/TES/DS/DS_19_12_18/DS_19_12_18.tex

367 lines
17 KiB
TeX

\documentclass[a4paper,10pt]{article}
\usepackage{myXsim}
%\usepackage[inline]{enumitem}
%\usepackage{tasks}
\title{DS 4}
\tribe{Terminale L-ES}
\date{20 décembre 2019}
\xsimsetup{
solution/print = false
}
\begin{document}
\maketitle
Le barème est donné à titre indicatif, il pourra être modifié.
Une part importante de la note sera dédiée à la rédaction, aux explications et à l'utilisation des notations mathématiques.
\begin{exercise}[subtitle={Gestion d'un parc de vélos}]
Afin de conserver au fil des années un parc en bon état, un loueur de vélos se sépare chaque hiver de 20\,\% de son stock et achète ensuite $35$ nouveaux vélos.
On modélise la situation par une suite $\left(u_n\right)$ où, pour tout entier naturel $n$,\: $u_n$ représente le nombre de vélos présents dans le stock de ce loueur au 1\up{er} juillet de l'année $(2018 + n)$.
Au 1\up{er} juillet 2018, le loueur possède $150$ vélos, ainsi $u_0 = 150$.
\medskip
\begin{enumerate}
\item
\begin{enumerate}
\item Déterminer le nombre de vélos dans le stock du loueur au 1\up{er} juillet 2019.
\item Justifier que, pour tout entier naturel $n$, on a : $u_{n+1} = 0,8u_n + 35$.
\end{enumerate}
\item On a calculé les premiers termes de cette suite à l'aide d'un tableur.
Une copie d'écran est donnée ci-dessous:
\begin{center}
\begin{tabularx}{0.8\linewidth}{|*{3}{>{\centering \arraybackslash}X|}}\hline
&A &B\\ \hline
1 &rang $n$ &terme $u_n$\\ \hline
2 &0 &150 \\ \hline
3 &1 &155\\ \hline
4 &2 &159 \\ \hline
5 &3 &162,2\\ \hline
\end{tabularx}
\end{center}
\begin{enumerate}
\item Quelle formule peut-on saisir dans la cellule B3 pour obtenir, par copie vers le bas, les termes successifs de la suite $\left(u_n\right)$ ?
\item Pour les termes de rang 36, 37, 38, 39 et 40, on obtient les résultats suivants (arrondis au millième) :
\begin{center}
\begin{tabularx}{0.8\linewidth}{|*{3}{>{\centering \arraybackslash}X|}}\hline
38& 36& 174,992 \\ \hline
39& 37& 174,994\\ \hline
40& 38& 174,995\\ \hline
41& 39& 174,996\\ \hline
42& 40& 174,997\\ \hline
\end{tabularx}
\end{center}
Conjecturer la limite de la suite $\left(u_n\right)$.
\end{enumerate}
\item \emph{Dans cette question, on cherche à démontrer la conjecture émise à la question précédente}.
Pour cela, on pose pour tout entier naturel $n$ : $v_n= u_n - 175$.
\begin{enumerate}
\item Démontrer que la suite $\left(v_n\right)$ est une suite géométrique dont on précisera la raison et le premier terme.
\item En déduire que, pour tout entier naturel $n$, on a : $u_n = - 25 \times 0,8^n + 175$.
\item Déterminer alors la limite de la suite $\left(u_n\right)$.
\end{enumerate}
\item On admet que la suite $\left(u_n\right)$ est croissante. On veut déterminer la plus petite valeur de $n$ tels que : $u_n \geqslant 170$.
\begin{enumerate}
\item Compléter l'algorithme en annexe pour trouver cette valeur.
\item Exécuter cet algorithme pour trouver la valeur de $n$. Interpréter le résultat dans le contexte de l'exercice.
\end{enumerate}
\end{minipage}
\end{enumerate}
\end{exercise}
\begin{solution}
\begin{enumerate}
\item
\begin{enumerate}
\item $u_1=150\times 0,8+35=155$. Au 1\up{er} juillet 2019, le loueur aura 155 vélos.
\item Le terme $u_n$ correspond au nombre de vélos l'année $(2018+n)$, $u_{n+1}$ le nombre de vélos l'année suivante. D'une année à l'autre il vend 20 \% de son stock, il lui en reste donc 80 \% soit $0,8\times u_n$. Puis il ajoute 35 nouveaux vélos. Donc il aura l'année suivante $0,8\times u_n+35$.
Ainsi $\forall n\in \N$, $u_{n+1}=0,8u_n+35$
\end{enumerate}
\item
\begin{enumerate}
\item Dans la cellule $B3$, il faut saisir : $=0,8*\text{B}2+35$
\item Le tableau donnant les termes de la suite pour $n$ allant de 38 à 42 permet de conjecturer que\\
$\lim\limits_{n \rightarrow +\infty} u_n=175$
\end{enumerate}
\item On pose $\forall n\in \N$, $v_n=u_n-175$
\begin{enumerate}
\item $\forall n\in \N$, $v_{n+1}=u_{n+1}-175=0,8u_n+35-175=0,8u_n-140=0,8\left(u_n-\dfrac{140}{0,8}\strut\right)=0,8(u_n-175)=0,8v_n$\\
Donc la suite $(v_n)$ est la suite géométrique de raison $q=0,8$ et de premier terme $v_0=u_0-175=-25$
\item $\forall n\in \N$, $v_n=v_0\times q^n=-25\times 0,8^n$.
De plus $\forall n\in \N$, $u_n=v_n+175=-25\times 0,8^n+175$.
\item La suite géométrique $(v_n)$ a pour raison $q=0,8$. $q\in]-1\,;\,1[$ donc $\displaystyle\lim_{n \to + \infty} 0,8^n = 0$ et $\lim\limits_{n \rightarrow +\infty} v_n=0$.
$\lim\limits_{n \rightarrow +\infty} u_n=\lim\limits_{n \rightarrow +\infty} v_n+175=0+175=175$.
\end{enumerate}
\item
\begin{enumerate}
\item ~
\begin{center}
\begin{minipage}{0.5\linewidth}
\begin{algorithm}[H]
\SetAlgoLined
$u \leftarrow 150$ \;
$n \leftarrow 0$ \;
\Tq{$n < 179$}{
$u \leftarrow u*0.9+35$ \;
$n \leftarrow n+1$ \;
}
\end{algorithm}
\end{minipage}
\end{center}
\item
En tâtonnant avec la calculatrice, on obtient
\[
u_7 = 169,76 \qquad \qquad u_8 = 170,8
\]
Donc au bout de 8 années, soit le 1\up{er} juillet 2026, le loueur possèdera plus de 170 vélos dans son stock.
\end{enumerate}
\end{enumerate}
\end{solution}
\pagebreak
\begin{exercise}[subtitle={Club de foot}]
Un club de football est composé d'équipes adultes masculines, adultes féminines et d'équipes d'enfants. Chaque week-end, la présidente Claire assiste au match d'une seule des équipes du club et elle suit:
\begin{list}{\textbullet}{}
\item dans 10\,\% des cas, le match d'une équipe adulte féminine;
\item dans 40\,\% des cas, le match d'une équipe adulte masculine;
\item dans les autres cas, le match d'une équipe d'enfants.
\end{list}
Lorsqu'elle assiste au match d'une équipe masculine, la probabilité que celle-ci gagne est $0,6$. Lorsqu'elle assiste au match d'une équipe d'enfants, la probabilité que celle-ci gagne est $0,54$.
La probabilité que Claire voie l'équipe de son club gagner est $0,58$.
On choisit un week-end au hasard. On note les évènements suivants:
\begin{list}{\textbullet}{}
\item $F$: \og Claire assiste au match d'une équipe adulte féminine \fg{};
\item $M$: \og Claire assiste au match d'une équipe adulte masculine \fg{};
\item $E$: \og Claire assiste au match d'une équipe d'enfants \fg{};
\item $G$: \og l'équipe du club de Claire gagne le match \fg{}.
\end{list}
\smallskip
\emph{Pour tous évènements $A$ et $B$, on note $\overline{A}$ l'évènement contraire de $A$, $p(A)$ la probabilité de $A$ et, si $B$ est de probabilité non nulle, $p_{B}(A)$ la probabilité de $A$ sachant $B$.}
\medskip
\begin{enumerate}
\item L'arbre de probabilité est donné en \textbf{annexe}. Le compléter au fur et à mesure de l'exercice.
\item Déterminer la probabilité $p(M \cap G)$.
\item
\begin{enumerate}
\item Démontrer que $p(F \cap G) = 0,07$.
\item En déduire $p_{F}(G)$.
\item La probabilité que l'équipe adulte féminine gagne un match est $0,47$. La présence de Claire semble-t-elle favoriser la victoire de l'équipe adulte féminine?
\end{enumerate}
\item Claire annonce avoir assisté à la victoire d'une équipe du club. Quelle est la probabilité qu'elle ait suivi le match d'une équipe adulte féminine ?
\end{enumerate}
\end{exercise}
\begin{solution}
\begin{enumerate}
\item ~
\begin{center}
\begin{tikzpicture}[grow=right, xscale=2]
\node {.}
child {node {$E$}
child {node {$G$}
edge from parent
node[above] {0.54}
}
child {node {$\overline{G}$}
edge from parent
node[above] {0.46}
}
edge from parent
node[above] {0.5}
}
child[missing] {}
child {node {$M$}
child {node {$G$}
edge from parent
node[above] {0.6}
}
child {node {$\overline{G}$}
edge from parent
node[above] {0.4}
}
edge from parent
node[above] {0.4}
}
child[missing] {}
child { node {$F$}
child {node {$G$}
edge from parent
node[above] {0.7}
}
child {node {$\overline{G}$}
edge from parent
node[above] {0.3}
}
edge from parent
node[above] {0.1}
} ;
\end{tikzpicture}
\end{center}
\item $p(M \cap G)= p(M)\times p_{M}(G)=0,4 \times 0,6=0,24$
\item
\begin{enumerate}
\item D'après la formule des probabilités totales:
$p(G)=p(F\cap G) + p(M\cap G)+p(E\cap G)$.
On sait que $p(G)=0,58$ et que $p(M\cap G)=0,24$.
\[
p(E \cap G)= p(E)\times p_{E}(G)=0,5 \times 0,54=0,27
\]
On en déduit que $p(F \cap G) = p(G) - p(M\cap G) - p(E\cap G) = 0,58-0,24-0,27 = 0,07$.
\item $p(F \cap G) = p(F)\times p_{F}(G)$ donc
\[
p_{F}(G) = \dfrac{p(F \cap G)}{p(F)} = \dfrac{0,07}{0,1}=0,7
\]
On peut ainsi compléter l'arbre (voir \textbf{annexe)}.
\item La probabilité que l'équipe adulte féminine gagne un match est $0,47$.% La présence de Claire semble-t-elle favoriser la victoire de l'équipe adulte féminine?
La probabilité que l'équipe féminine gagne un match sachant que Claire a assisté au match est
$p_{F}(G) = 0,7$.
Donc la présence de Claire semble favoriser la victoire de l'équipe féminine.
\end{enumerate}
\item Claire annonce avoir assisté à la victoire d'une équipe de club.
La probabilité qu'elle ait suivi le match d'une équipe adulte féminine est
$p_{G}(F)=\dfrac{p(F\cap G)}{p(G)} = \dfrac{0,07}{0,58} \approx 0,12$.
\end{enumerate}
\end{solution}
\begin{exercise}[subtitle={Vrai ou Faux}]
Pour chacune des affirmations suivantes, indiquer si elle est vraie ou fausse et justifier la réponse donnée.
\begin{enumerate}
\item On considère l'arbre pondéré suivant:
\begin{center}
\begin{tikzpicture}[xscale=2, grow=right]
\node {.}
child {node {$R$}
child {node {$S$}
edge from parent
node[below] {0.4}
}
child {node {$\overline{S}$}
edge from parent
node[above] {0.6}
}
edge from parent
node[below] {0.7}
}
child[missing] {}
child { node {$\overline{R}$}
child {node {$S$}
edge from parent
node[below] {0.2}
}
child {node {$\overline{S}$}
edge from parent
node[above] {0.8}
}
edge from parent
node[above] {0.3}
} ;
\end{tikzpicture}
\end{center}
\textbf{Affirmation 1}: La probabilité de $\overline{R}$ sachant $S$ est $0,06$.
\medskip
\item Soit $k$ un réel tel que $0 \leqslant k < 18$. Soit $X$ une variable aléatoire qui suit la loi uniforme sur l'intervalle $\left [k~;\, 18\strut\right ]$. On suppose que l'espérance de $X$ est égale à 12.
\smallskip
\textbf{Affirmation 2}: La valeur de $k$ est 9.
\medskip
\item Soit $f$ une fonction dérivable sur l'intervalle $\left [0~;\, 15\strut\right ]$. On suppose que sa fonction dérivée, notée $f'$, est continue sur $\left [0~;\, 15\strut\right ]$. Les variations de $f'$ sont représentées dans le tableau ci-dessous.
\begin{minipage}{0.4\linewidth}
\begin{tikzpicture}[baseline=(a.north)]
\tkzTabInit[lgt=1,espcl=2]{$x$/1,$f'(x)$/2}{0, 5, 15}
\tkzTabVar{+/ 30, -/ -5, +/ 10}
\end{tikzpicture}
\end{minipage}
\begin{minipage}{0.6\linewidth}
\textbf{Affirmation 3}: La courbe représentative $\mathcal{C}_f$ de la fonction $f$ admet une et une seule tangente parallèle à l'axe des abscisses.
\bigskip
\textbf{Affirmation 4}: La fonction $f$ est convexe sur $\left [5~;\, 15\strut\right ]$.
\end{minipage}
\end{enumerate}
\end{exercise}
\begin{solution}
\begin{enumerate}
\item On a $P_S\left(\overline{R}\right) = \dfrac{P\left(S \cap \overline{R}\right)}{P(S)}$.
$\bullet~~$ $P\left(S \cap \overline{R}\right) = P\left(\overline{R} \cap S\right) = P\left(\overline{R} \right) \times P_{\overline{R}}(S) = 0,3 \times 0,2 : 0,06$.
$\bullet~~$ D'après la loi des probabilités totales :
$P(S) = P(R \cap S) + P\left(\overline{R} \cap S \right) = P(R) \times P_R (S) + P\left(\overline{R} \right) \times P_{\overline{R}}(S) = 0,7 \times 0,4 + 0,3 \times 0,2 = 0,28 + 0,06 = 0,34$.
Donc $\dfrac{P\left(S \cap \overline{R}\right)}{P(S)} = \dfrac{0,6}{0,34} \approx 0,18 \neq 0.06$ : l'affirmation est fausse.
\medskip
\item L'espérance de $X$sur $[k~;~18]$ est égale à $\dfrac{k + 18}{2} = 12 \iff k +18 = 24 \iff k = 6$ : l'affirmation est fausse.
\medskip
\item \smallskip
\textbf{Affirmation 3}: La courbe représentative $\mathcal{C}_f$ de la fonction $f$ admet une et une seule tangente parallèle à l'axe des abscisses.
\smallskip
D'après le tableau de variations de $f'$, cette dérivée s'annule sur l'intervalle [0~;~5] et sur l'intervalle [5~;~15]. Il existe donc $a \in [0~;~5[$ tel que $f'(a) = 0$ et $b \in ]5~;~15]$ tel que $f'(b) = 0$. En ces deux points distincts le nombre dérivé est nul ce qui signifie que les tangentes à la courbe $\mathcal{C}_f$ sont horizontales : l'affirmation est fausse.
\textbf{Affirmation 4}: La fonction $f$ est convexe sur $\left [5~;\, 15\strut\right ]$.
D'après le tableau de variations $f'$croissante sur $[5~;~15]$ donc la fonction $f$ est convexe sur cet intervalle : affirmation vraie.
\end{enumerate}
\end{solution}
\end{document}
%%% Local Variables:
%%% mode: latex
%%% TeX-master: "master"
%%% End: